Daniel's symmetric sum

Algebra Level 5

Consider the set S = { 1 , 2 , 3 , , 101 } S=\{1,2,3,\cdots,101\} , and let S k S_k be the sum of the product of every k k element subset of S S (also called the k k th symmetric sum of S S ). Find the remainder when k = 1 101 S k \sum_{k=1}^{101} S_k is divided by 101 101 .

This problem is posed by Daniel C .

Details and assumptions

You may use the fact that 101 is prime.


The answer is 100.

This section requires Javascript.
You are seeing this because something didn't load right. We suggest you, (a) try refreshing the page, (b) enabling javascript if it is disabled on your browser and, finally, (c) loading the non-javascript version of this page . We're sorry about the hassle.

14 solutions

Jing Hao Pang
Nov 24, 2013

Full Solution

Note that:

k = 1 101 S k = ( 1 + 1 ) ( 2 + 1 ) ( 3 + 1 ) ( 100 + 1 ) ( 101 + 1 ) 1 \displaystyle \sum_{k=1}^{101} S_{k} = (1+1)(2+1)(3+1) \ldots (100+1)(101+1)-1

0 1 ( m o d 101 ) 100 ( m o d 101 ) \equiv 0-1 \pmod{101} \equiv 100 \pmod{101}

Therefore 100 \boxed{100} is the answer.

Motivation and thought process

Since this is a huge expression, I immediately thought of factorising the expression. Instead of having numbers in set S S , I decided to replace them with S = { a 1 , a 2 , , a 101 } S=\left\{a_{1},a_{2}, \ldots ,a_{101}\right\} .

At first I wasn't sure how to begin factorising, so I decided to try simpler versions of the summation:

S = { a 1 , a 2 } , k = 1 2 S k = a 1 + a 2 + a 1 a 2 = ( a 1 + 1 ) ( a 2 + 1 ) 1 S= \left\{a_{1},a_{2}\right\}, \displaystyle \sum_{k=1}^{2} S_{k} = a_{1}+a_{2}+a_{1}a_{2}=(a_{1}+1)(a_{2}+1)-1

S = { a 1 , a 2 , a 3 } , k = 1 3 S k = a 1 + a 2 + a 3 + a 1 a 2 + a 2 a 3 + a 3 a 1 + a 1 a 2 a 3 S= \left\{a_{1},a_{2},a_{3}\right\}, \displaystyle \sum_{k=1}^{3} S_{k} = a_{1}+a_{2}+a_{3}+a_{1}a_{2}+a_{2}a_{3}+a_{3}a_{1}+a_{1}a_{2}a_{3}

= ( a 1 + 1 ) ( a 2 + 1 ) ( a 3 + 1 ) 1 =(a_{1}+1)(a_{2}+1)(a_{3}+1)-1

Why can it be factorised in such a way? This question can be answered using the degrees of the terms. For example, if we wanted the sum of all the terms with degree 2 2 ( S 2 S_{2} ), we simply choose two a a values, let's say a 1 a_{1} and a 2 a_{2} , and choose 1 1 s for all other terms, obtaining a degree 2 term. Repeating for every single pair of a a values and taking the sum, we end up with S 2 S_{2} . This can be repeated for every degree. Since the summation does not have 1 1 , we put a 1 -1 at the end of the product.

Thus, returning back to the question, it is now clear that the summation can be factorised into

( a 1 + 1 ) ( a 2 + 1 ) ( a 3 + 1 ) . . . ( a 101 + 1 ) 1 (a_{1}+1)(a_{2}+1)(a_{3}+1)...(a_{101}+1)-1

Subbing a 1 = 1 a_{1} = 1 , a 2 = 2 a_{2} = 2 , etc., we get

( 1 + 1 ) ( 2 + 1 ) ( 3 + 1 ) ( 100 + 1 ) ( 101 + 1 ) 1 (1+1)(2+1)(3+1) \ldots (100+1)(101+1)-1 .

Generalisation

For set S = { a 1 , a 2 , , a n } S=\left\{a_{1}, a_{2}, \ldots ,a_{n} \right\} , k = a 1 a n S k = ( a 1 + 1 ) ( a 2 + 1 ) ( a 3 + 1 ) . . . ( a n + 1 ) 1 \displaystyle \sum_{k={a_1}}^{a_n} S_{k} = (a_{1}+1)(a_{2}+1)(a_{3}+1)...(a_{n}+1)-1

Great generalisation, so we didn't needed Wilson or that 101 is prime after all.

Yong See Foo - 7 years, 6 months ago

Oops, my generalisation is slightly off. I'll re-write it here.

For set S = { a 1 , a 2 , , a n } S = \left\{a_{1},a_{2}, \ldots , a_{n} \right\} , where a a is a real number,

k = 1 n S k = ( a 1 + 1 ) ( a 2 + 1 ) ( a 3 + 1 ) ( a n + 1 ) 1 \displaystyle \sum_{k=1}^{n} S_{k} = (a_{1}+1)(a_{2}+1)(a_{3}+1) \ldots (a_{n}+1)-1

Jing Hao Pang - 7 years, 6 months ago

Log in to reply

We know that k = 1 101 S k 100 ( m o d 101 ) \sum_{k=1}^{101} S_k \equiv 100 \pmod{101} . Could you find a method to find the residue mod 101 of a particular S i S_i , say S 30 ( m o d 101 ) S_{30} \pmod{101} ?

Jorge Tipe - 7 years, 6 months ago

Log in to reply

May I clarify whether set S S must strictly be { 1 , 2 , 3 , , 101 } \left\{1,2,3,\ldots,101\right\} or can the elements in the set change?

Jing Hao Pang - 7 years, 6 months ago

Log in to reply

@Jing Hao Pang Consider S = { 1 , 2 , , 101 } S=\{1, 2, \ldots, 101\}

Jorge Tipe - 7 years, 6 months ago

wow!!!

Rindell Mabunga - 7 years, 6 months ago
Snehal Shekatkar
Nov 25, 2013

Motivation: We see all possible k k products in the problem, which reminds us of Vieta's formulae . So let us try to think in terms of polynomial.

Now consider a monic polynomial p ( x ) p(x) of degree 101 101 whose roots are 1 , 2 , 3 , . . . , 101 {1,2,3,...,101} . Clearly,

p ( x ) = i = 1 i = 101 ( x i ) p(x)=\prod_{i=1}^{i=101} (x-i)

We also see that in the expanded form, we can write p ( x ) p(x) as:

p ( x ) = k = 0 101 a k x 101 k p(x)=\sum_{k=0}^{101}a_{k}x^{101-k}

and a 0 = 1 a_{0}=1

Using Vieta's relations for p ( x ) p(x) , we get,

a k = ( 1 ) k S k a_{k}=(-1)^{k}S_{k}

Also, using expanded form of p ( x ) p(x) , we get,

p ( 1 ) = 1 k = 1 101 S k p(-1)=-1-\sum_{k=1}^{101}S_{k}

k = 1 101 S k = 1 p ( 1 ) \therefore \sum_{k=1}^{101}S_{k}=-1-p(-1)

Using product form of p ( x ) p(x) , we find that,

p ( 1 ) = ( 2 ) × ( 3 ) × . . . × ( 102 ) = 102 ! p(-1)=(-2)\times(-3)\times...\times(-102)=-102!

Hence, we get,

k = 1 101 S k = 102 ! 1 \sum_{k=1}^{101}S_{k}=102!-1

This shows that we need to subtract 100 100 from k = 1 101 S k \sum_{k=1}^{101}S_{k} so that resulting number would be divisible by 101 101 .

Hence the required remainder is: 100 \boxed{100}

wht is mod101

Ashwinikumar Gupta - 7 years, 6 months ago

Log in to reply

What is the meaning of your question? Do you not know meaning of mod or is there some other issue?

Snehal Shekatkar - 7 years, 6 months ago
Zi Song Yeoh
Nov 24, 2013

Let f ( x ) = ( x + 1 ) ( x + 2 ) . . . ( x + 101 ) f(x) = (x + 1)(x + 2)...(x + 101) . We note that f ( 1 ) f(1) is the sum of coefficient of the polynomial. Also, note that the required sum is equal to the sum of coefficients of f ( x ) f(x) minus 1 1 . (Indeed, it is easy to see this by expanding the polynomial or Vieta's formula.) Let S S be the required sum. Then, S = f ( 1 ) 1 = 102 ! 1 1 100 ( m o d 101 ) S = f(1) - 1 = 102! - 1 \equiv -1 \equiv \boxed{100} \pmod{101} .

We know that k = 1 101 S k 100 ( m o d 101 ) \sum_{k=1}^{101} S_k \equiv 100 \pmod{101} . Could you find a method to find the residue mod 101 of a particular S i S_i , say S 30 ( m o d 101 ) S_{30} \pmod{101} ?

Jorge Tipe - 7 years, 6 months ago

Log in to reply

Yes. For p p prime we get the set 1 , 2 , . . . , p 1,2,...,p , and for 1 i < p 1\le i<p we get S i 0 ( m o d p ) S_i\equiv 0\pmod p . It remains to evaluate S p 1 ( p 1 ) ! 1 ( m o d p ) S_{p-1}\equiv (p-1)!\equiv -1\pmod p .

Arkan Megraoui - 7 years, 6 months ago

Log in to reply

PS: Can you show why S i 0 ( m o d p ) S_i\equiv 0\pmod p for 1 i < p 1\le i<p ??? :)

Arkan Megraoui - 7 years, 6 months ago

Log in to reply

@Arkan Megraoui You can read the solution of Mark Liu, OR use the following idea: In Z p [ x ] \mathbb{Z}_p[x] (the set of polymomials with coefficients in Z p \mathbb{Z}_p ) the polynomial x p x x^p-x has (distinct) roots 1 , 2 , , p 1, 2, \ldots, p then x p x = ( x 1 ) ( x 2 ) . . . ( x p ) \overline{x^p-x}=\overline{(x-1)(x-2)...(x-p)} , where f ( x ) = g ( x ) \overline{f(x)}=\overline{g(x)} means that the polynomials f ( x ) f(x) and g ( x ) g(x) have their corresponding coefficients congruent modulo p p .

Jorge Tipe - 7 years, 6 months ago
Mark Liu
Nov 24, 2013

We know the order of 2 must divide 100. We can check by hand that none of 2 , 2 2 , 2 5 , 2 10 , 2 20 , 2 50 2, 2^2, 2^5, 2^{10}, 2^{20}, 2^{50} are congruent to 1. Therefore 2 is a primitive root and 2 k 1 2^k \ncong \ 1 for 1 k < 100 1 \le k \lt 100

Now note that 2 k S k S k 2^k S_k \equiv \ S_k since multiplying by 2 is a permutation.
So S k ( 2 k 1 ) 0 S^k ( 2^k - 1) \equiv 0 and when k < 100 k \lt 100 then 2 k 1 2^k \ncong 1 hence S k 0 S_k \equiv 0 for k < 100 k \lt 100

Then S 100 = 1 S_{100} = -1 by Wilson's theorem, and the result follows.

Motivation: I saw that multiplying the first symmetric sum by 2 (or any other element) just permutes every term and therefore leaves the sum fixed. This implies that the first symmetric sum is 0. Then I extended the argument. And actually the 2 is irrelevant if we appeal to the existence of a primitive root, it just so happens that the primitive is 2 in this case.

Mark Liu - 7 years, 6 months ago

Log in to reply

Since your solution is different from the others, could you explain in more details the second paragraph of your solution?

Jorge Tipe - 7 years, 6 months ago

Log in to reply

Okay let's do a smaller case. Suppose we are doing this question for mod 5 and we have found that 2 is again, a primitive root. Let's calculate S 2 = 1 2 + 1 3 + 1 4 + 2 3 + 2 2 + 2 3 + 2 4 + 3 4 S_2 = 1*2 + 1*3 + 1*4 + 2*3 + 2*2 + 2*3 + 2*4 + 3*4 Note that if we listed every element out the elements {1, 2, 3, 4} and then multiplied each one by 2, we would get {2, 4, 1, 3} which is the same set! And since the symmetric sums depend only on the definition of the set, we can multiply the set by 2 and then take the symmetric sum again and get the same result. S 2 = ( 2 1 ) ( 2 2 ) + ( 2 1 ) ( 2 3 ) + ( 2 1 ) ( 2 4 ) + ( 2 2 ) ( 2 3 ) + ( 2 2 ) ( 2 2 ) + ( 2 2 ) ( 2 3 ) + ( 2 2 ) ( 2 4 ) + ( 2 3 ) ( 2 4 ) 2 2 ( S 2 ) S_2 = (2*1)*(2*2) + (2*1)*(2*3) + (2*1)*(2*4) + (2*2)*(2*3) + (2*2)*(2*2) + (2*2)*(2*3) + (2*2)*(2*4) + (2*3)*(2*4) \equiv 2^2(S_2) Is it any clearer?

Mark Liu - 7 years, 6 months ago

Log in to reply

@Mark Liu Nice solution Mark, thanks for your explanation!

Jorge Tipe - 7 years, 6 months ago

In fact, note that for $p$ prime, k = 1 p S k k = 1 p 1 S k ( m o d p ) \sum_{k=1}^p S_k\equiv \sum_{k=1}^{p-1} S_k\pmod p .

Arkan Megraoui - 7 years, 6 months ago
Kee Wei Lee
Nov 24, 2013

At first the notations are really scary, so it is good to work with smaller cases first. Suppose we have a smaller set called N = { 1 , 2 , 3 , 4 } N=\{1,2,3,4\} If so by defination we get that;

N 1 = 1 + 2 + 3 + 4 N_1=1+2+3+4

N 2 = 1 2 + 1 3 + 1 4 + 2 3 + 2 4 + 3 4 N_2=1*2+1*3+1*4+2*3+2*4+3*4

N 3 = 1 2 3 + 1 2 4 + 1 3 4 + 2 3 4 N_3=1*2*3+1*2*4+1*3*4+2*3*4

N 4 = 1 2 3 4 N_4=1*2*3*4

So looking at this, we see that N k N_k somehow relates to Vieta's theorem. At which N k N_k repersents the x n k x^{n-k} coefficient of the monic polynomial with roots 1 1 to n n .

More generally if the set is X = { a 1 , a 2 , . . . , a n 1 , a n } X=\{a_1,a_2,...,a_{n-1},a_n\} Then;

X 1 = a 1 + a 2 + . . . + a n 1 + a n X_1=a_1+a_2+...+a_{n-1}+a_n

X 2 = a 1 a 2 + a 1 + a 3 + . . . + a n 2 a n 1 + a n 1 a n X_2=a_1*a_2+a_1+a_3+...+a_{n-2}*a_{n-1}+a_{n-1}*a_{n}

. . . ...

X k = a 1 a 2 . . . a k + a 1 a 2 . . . a k + 1 + . . . + a n k + 1 . . . a n X_k=a_1*a_2*...*a_{k}+a_1*a_2*...*a_{k+1}+...+a_{n-k+1}*...*a_n

. . . ...

X n = a 1 a 2 . . . a n X_n=a_1*a_2...*a_n

So we can conclude that,

( x a 1 ) ( x a 2 ) . . . ( x a n ) = x n + ( 1 ) 1 X 1 x n 1 + ( 1 ) 2 X 2 x n 2 + . . . . + ( 1 ) n 1 X n 1 x + ( 1 ) n X n (x-a_1)(x-a_2)...(x-a_n)=x^n+(-1)^1X_1x^{n-1}+(-1)^2X_2x^{n-2}+....+(-1)^{n-1}X_{n-1}x+(-1)^{n}X_n

by Vieta's theorem.

So we have the set S = { 1 , 2 , 3... , 101 } S=\{1,2,3...,101\} , applying this to the polynomial we just found,

( x 1 ) ( x 2 ) . . . ( x 101 ) = x 101 S 1 x 100 + . . . + S 100 x S 101 (x-1)(x-2)...(x-101)=x^{101}-S_1x^{100}+...+S_{100}x-S_{101}

So to find k = 1 101 S k \displaystyle \sum_{k=1}^{101} S_k we let x = 1 x=-1

( 1 1 ) ( 1 2 ) . . . ( 1 101 ) = ( 1 ) 101 S 1 ( 1 ) 100 + . . . S 100 ( 1 ) S 101 (-1-1)(-1-2)...(-1-101)=(-1)^{101}-S_1(-1)^{100}+...S_{100}(-1)-S_{101}

102 ! = 1 S 1 S 2 . . . S 100 S 101 \Rightarrow -102!=-1-S_1-S_2-...-S_{100}-S_{101}

102 ! = 1 + S 1 + S 2 + . . . + S 100 + S 101 \Rightarrow 102!=1+S_1+S_2+...+S_{100}+S_{101}

k = 1 101 S k = 102 ! 1 \Rightarrow \displaystyle \sum_{k=1}^{101} S_k =102!-1

If so, we know that 102 ! 0 m o d 101 102! \equiv 0 \mod 101

Therefore,

102 ! 1 1 m o d 101 102!-1 \equiv -1 \mod 101

or 102 ! 1 100 m o d 101 102!-1 \equiv 100 \mod 101

Hence k = 1 101 S k 100 m o d 101 \displaystyle \sum_{k=1}^{101} S_k \equiv \boxed{100} \mod101

What is mod 101 ?

Krishna Gundu - 7 years, 6 months ago

Log in to reply

Remainder when you divide by 101. For example, 5 4 ( m o d 101 ) 5\equiv 4\pmod {101} , and 113 8 ( m o d 101 ) 113\equiv 8\pmod {101} . For the more interesting properties of modular arithmetic, I highly recommend the first chapter of "Disquisitiones Arithmeticae" by the great Gauss. ; )

Arkan Megraoui - 7 years, 6 months ago
Marek Bernat
Nov 25, 2013

We use the fact that symmetric sums are very close to polynomials. For example to solve a quadratic equation, one might use that ( x a ) ( x b ) = x 2 ( a + b ) x + a b (x-a)(x-b) = x^2 - (a+b)x + ab to relate the roots with the coefficients of the equation. Here a + b a+b is the first symmetric sum and a b ab the second symmetric sum. Let's move one dimension up to see the pattern ( x + x 1 ) ( x + x 2 ) ( x + x 3 ) = x 3 + x 2 S 1 + x S 2 + S 3 (x + x_1) (x + x_2) (x + x_3) = x^3 + x^2 \cdot S_1 + x \cdot S_2 + S_3 where S 1 = x 1 + x 2 + x 3 S_1 = x_1 + x_2 + x_3 S 2 = x 1 x 2 + x 2 x 3 + x 3 x 1 S_2 = x_1 x_2 + x_2 x_3 + x_3 x_1 and S 3 = x 1 x 2 x 3 S_3 = x_1 x_2 x_3 . It's straightforward to generalize this to any dimension as i = 1 n ( x + x i ) = i = 0 n x n i S i \prod_{i=1}^n (x + x_i) = \sum_{i=0}^n x^{n-i}\cdot S_i where we used S 0 = 1 S_0 = 1 for convenience.

To get back to the original problem, notice that when we put n = 101 n = 101 , x = 1 x = 1 and x i = i x_i = i we get almost the sum in the question. k = 1 101 S k = k = 1 101 ( 1 + k ) 1. \sum_{k=1}^{101} S_k = \prod_{k=1}^{101} (1 + k) - 1. But the product collapses because it also contains k = 100 k = 100 and 1 + 100 0 ( m o d 101 ) 1 + 100 \equiv 0 \pmod{101} and so the answer is 1 -1 or, more neatly, 100 \bf 100 .

Will Song
Nov 25, 2013

Consider the polynomial P ( X ) = ( X + 1 ) ( X + 2 ) ( X + 101 ) = X 101 + S 1 X 100 + + S 101 P(X) = (X + 1)(X + 2) \cdots (X + 101) = X^{101} + S_1 X^{100} + \cdots + S_{101} Pluggin in P ( 1 ) P(1) , we get 1 + S 1 + + S 101 = 102 ! 0 ( m o d 101 ) 1 + S_1 + \cdots + S_{101} = 102! \equiv 0 \pmod{101} , so the answer is 100 100

Erick Wong
Nov 25, 2013

The fact that 101 is prime is irrelevant. If we add 1 to the sum, we get the sum of all products of all subsets of S, which factors cleanly into x S ( 1 + x ) \prod_{x \in S} ( 1 + x) (we see easily that each subset's product occurs exactly once when this is expanded). This is divisible by 101 since 100 is in S. Therefore the original sum is congruent to -1 mod 101, i.e. 100.

Michael Tang
Nov 25, 2013

Symmetric sums remind me of Vieta's formulas, which express the symmetric sums in terms of the coefficients of a polynomial. So, let

f ( x ) = ( x 1 ) ( x 2 ) ( x 101 ) . (1) \qquad f(x) = (x-1)(x-2)\ldots(x-101). \qquad \textbf{(1)}

Expanding, we get f ( x ) = x 101 ( 1 + 2 + + 101 ) x 100 + 1 2 101 = x 101 S 1 x 100 + S 2 x 99 + S 100 x S 101 . \begin{aligned} f(x) &= x^{101} - (1+2+\ldots + 101) x^{100} + \ldots - 1\cdot2\cdots101 \\ &= x^{101} - S_1 x^{100} + S_2 x^{99} - \ldots + S_{100} x - S_{101}. \end{aligned}

Great! We have an expression containing every symmetric sum. Now, we want to set x x equal to some number, to get the summation we want. The only problem, however, is that the signs of the sums are alternating. Luckily, we can fix this by setting x = 1 x = -1 :

f ( 1 ) = ( 1 ) 101 S 1 ( 1 ) 100 + S 2 ( 1 ) 99 + S 100 ( 1 ) S 101 = 1 S 1 S 2 S 100 S 101 = 1 k = 1 101 S k . \begin{aligned} f(-1) &= (-1)^{101} - S_1 (-1)^{100} + S_2 (-1)^{99} - \ldots + S_{100} (-1) - S_{101} \\ &= -1 - S_1 - S_2 - \ldots - S_{100} - S_{101} \\ &= -1 - \sum_{k = 1}^{101} S_k. \end{aligned}

(It is a well-known fact that for positive integers n , n, ( 1 ) n = 1 (-1)^n = 1 if n n is even, and ( 1 ) n = 1 (-1)^n = -1 if n n is odd. So, the powers of 1 -1 also alternate, which motivated this step.)

But we also know that

f ( 1 ) 0 ( m o d 101 ) , f(-1) \equiv 0 \pmod{101},

since in equation ( 1 ) , (1), f ( 1 ) f(-1) is a product of integers which contains the term 101. -101. Thus 1 k = 1 101 S k 0 ( m o d 101 ) -1 - \sum_{k = 1}^{101} S_k \equiv 0 \pmod{101} so k = 1 101 S k 100 ( m o d 101 ) . \sum_{k = 1}^{101} S_k \equiv \boxed{100} \pmod{101}.

Remark: Note that we never used the fact that 101 101 is prime; we can easily generalize this solution for any positive integer n , n, instead of 101. 101.

Another remark, after reading Zi Song's solution: To avoid having to plug in x = 1 , x = -1, we could use the polynomial f ( x ) = ( x + 1 ) ( x + 2 ) ( x + 101 ) f(x) = (x+1)(x+2)\ldots(x+101) instead. Cool solution, Zi Song!

Michael Tang - 7 years, 6 months ago
Sujoy Roy
Nov 24, 2013

We can prove that, ( x 1 ) ( x 2 ) ( x 101 ) = x 101 S 1 x 100 + S 2 x 99 S 3 x 98 + + S 100 x S 101 (x-1)(x-2) \ldots (x-101)=x^{101}-S_1x^{100}+S_2x^{99}-S_3x^{98}+\ldots+S_{100}x-S_{101} . Now put x = 1 x=-1 in the above identity we get, S 1 + S 2 + + S 101 = 2 3 102 1 S_1+S_2+\ldots+S_{101}=2*3*\ldots*102-1 . Now k = 1 101 S k ( m o d 101 ) = 1 ( m o d 101 ) = 100 \sum_{k=1}^{101}S_k (\mod 101)=-1(\mod 101)=100

Daniel Chiu
Nov 24, 2013

Consider the polynomial f ( x ) f(x) with the elements of S S as roots. This polynomial is f ( x ) = ( x 1 ) ( x 2 ) ( x 101 ) = x 101 + a 1 x 100 + + a 101 f(x)=(x-1)(x-2)\cdots(x-101)=x^{101}+a_1x^{100}+\cdots+a_{101} where the a 1 , a 2 , , a 101 a_1,a_2,\cdots,a_{101} are constants.

By Vieta's formulas, the answer is a 1 + a 2 a 3 + a 101 -a_1+a_2-a_3+\cdots-a_{101} However, notice that f ( 1 ) = 1 + a 1 a 2 + a 3 + a 101 f(-1)=-1+a_1-a_2+a_3-\cdots+a_{101} so the answer is f ( 1 ) 1 = ( 1 1 ) ( 1 2 ) ( 1 101 ) 1 = 102 ! 1 -f(-1)-1=-(-1-1)(-1-2)\cdots(-1-101)-1=102!-1 Finally, 102 ! 1 1 100 ( m o d 101 ) 102!-1\equiv -1\equiv \boxed{100}\pmod{101}

Motivation: Symmetric sums reminds me of Vieta's, and f ( 1 f(-1 being alternating signs on the coefficients is a handy trick to know.

Oops typo missed a right parenthesis on f ( 1 ) f(-1) .

Daniel Chiu - 7 years, 6 months ago
Shyan Akmal
Feb 7, 2014

We have that S 1 + S 2 + . . . + S 101 = ( 1 + 1 ) ( 1 + 2 ) ( 1 + 3 ) . . . ( 1 + 101 ) 1 = S_1+S_2+...+S_{101} = (1+1)(1+2)(1+3)...(1+101)-1= = 102 ! 1 1 100 m o d 101. =102! - 1\equiv -1\equiv 100\bmod 101.

Rik Ghosh
Nov 29, 2013

If we observe S1 ≡ 0(mod 101) , similarly S2 ≡ 0(mod 101) but only S100≡ -1(mod 101) since the product of the subset {1,2,3,4,…100} is ≡ -1(mod 101)

Santanu Banerjee
Nov 28, 2013

On careful observation we see that the values of Sk are the coefficients of the equation::

(x+1)(x+2)(x+3)........(x+101)

so to get the value of the summation part we put 1 (one) in the above expression and subtract 1(as the first term contains only x^101)

So we get (102! - 1) % 101

101 divides 102! but not 1 so we get the modulus as -1 (or the +ve mod as 100 (ANSWER ==100))

0 pending reports

×

Problem Loading...

Note Loading...

Set Loading...